Completed rec9 solutions
[course.git] / latex / problems / Young_and_Freedman_12 / problem28.18.tex
1 \begin{problem*}{28.18}
2 Two long, straight wires, one above the other, are seperated by a
3 distance $2a$ and are parallel to the $x$-axis.  Let the $+y$-axis be
4 in the plane of the wires in the direction from the lower wire to the
5 upper wire.  Each wire carries current $I$ in the $+x$-direction.
6 What are the magnitude and direction of the net magnetif field of the
7 two wires at a point in the plane of the wires \Part{a} midway between
8 them; \Part{b} at a distance $a$ above the upper wire; \Part{c} at a
9 distance $a$ below the lower wire?
10 \end{problem*}
11
12 \begin{solution}
13 \Part{a}
14 Using the right hand rules for magnetic field from a wire, we see that
15 the upper wire will create a magnetic field into the page while the
16 lower wire will create a magnetic field out of the page.  The
17 magnitude of the field from a single wire is given by
18 \begin{equation}
19   B = \frac{\mu_0 I}{2\pi r} \;,
20 \end{equation}
21 with the same current $I$ and distance $r=a$ for both wires.
22 Therefore the net magnetic field is \ans{zero}.
23
24 \Part{b}
25 Both wires create a magnetic field out of the page.  The magnitude of
26 the toal field will be
27 \begin{equation}
28   B = \frac{\mu_0 I}{2\pi a} + \frac{\mu_0 I}{2\pi (3a)}
29     = \ans{\frac{2 \mu_0 I}{3\pi a}} \;.
30 \end{equation}
31
32 \Part{c}
33 Both wires create a magnetic field into the page.  The magnitude of
34 the toal field will be
35 \begin{equation}
36   B = \frac{\mu_0 I}{2\pi (3a)} + \frac{\mu_0 I}{2\pi a}
37     = \ans{\frac{2 \mu_0 I}{3\pi a}} \;,
38 \end{equation}
39 the same as the magnitude for \Part{b}.
40 \end{solution}